Are these two triangles congruent?
Yes
No
Not enough information?

Are These Two Triangles Congruent?Yes No Not Enough Information?

Answers

Answer 1
no they are not congruent

Related Questions

What percent of the rectangle below is shaded (hint think about the fraction or ratio of
shaded to total)?
33%
40%
67%
25%

Answers

it is 33% i believe (it won’t let me send right so here’s some random words because it has to be 20 characters long)
the answer is a. 33%

what is it???????????

Answers

Answer:

4/5

Step-by-step explanation:

The rise is 4, and the run is 5. The rise comes first, so the answer is 4/5. The line is positive.

Answer:

slope = 4/5

Step-by-step explanation:

The best way to find the slope with just a line is to pick the most direct points on the graph, meaning points that are not in between numbers but right on the graph lines

y value - y value  over x value - x value  

Points: (-2,0) and (3,4)

4-0 = 4

3-(-2) = 5

4/5

Please only answer if you know

Answers

Answer:

A, C

Step-by-step explanation:

A.)

32 / 8 * 5.25 ----> 4 * 5.25 ----> 21

C.)

4 * 12.5 / 2----> 49 / 2 ----> 24.5

Answer:

between 20 & 30?

A & C  

Step-by-step explanation:

A) 32 / 8 * 5.25 = 21

B) -18 / 6 * 9 = -27

C) 4 * 12.5 / 2 = 25

D) 12 * 4[tex]\frac{1}{3}[/tex] / -2 = -26

Will give crown help

Answers

Answer:

n x 5

Step-by-step explanation:

The pattern is to multiply each number by 5 so multiplying n by 5 will get you the expression n x 5.

Hope that helps

I will give brainliest! In the figure, the circle has a center O and has a radius of 10. If the length of arc AB is between 5 and 6, what is one possible integer value of x?

Answers

Answer:

any integer in between 29,30,31,32,33,34 will work.

Plss help I’ll mark brainlist

Answers

Answer:

If it's 16, then it's -4 > x

Step-by-step explanation:

Ignore the inequality sign first and solve 12 = -3x

This would equal -4 = x

Now, when you divide or multiply both sides by a negative, you have to change the inequality sign so you change it from -4 > x to -4 < x

which of the equations below has a slope of 1/4

Answers

Answer:

uh you didnt put the picture please put the picture i really want to help you:)

Step-by-step explanation:

Answer:

bra

Step-by-step explanation:

Please help me with these questions ASAP!!!!!!!!

Answers

Answer:

Step-by-step explanation:

Answer:

For the first picture: m angle t=36 and m angle v=54

For the second picture:m angle P=55 m angle Q=81 and m angle QPR=180

Step-by-step explanation:

For the first picture, you had to do 2x+3x+90=180

We got the 90 from the right triangle.

Then you combine like terms, which was then 5x=90=180

Then, you subtract the 90 from both side, leaving 5x=90

Divide 90 by 5, which gives you x=18

For both the answers, you had to do 2(18) for T and 3(18) for V

The second picture, you did (8y + 1) + (5y + 5)= 136

You combine like terms, which levas you with 13y + 6 = 136

You subtract 6 from both sides, leaving you with 13y= 130

Divide 130 by 13, which is 10 so y=10

You then replace the y for (8y + 1) to 10 and solve

As well as for (5y + 5), you replace the y with 10 and solve

And m angle QRP is 180 because every triangle has a total of 180 degrees

Hope this helped!

what is the answer to this one need asap

Answers

Hello,

The correct answer is 18.
Hope this helps!!
Brainliest?

Given AABC : ADEF, find x.
E
B
x
8
А
С
D
12.
18
10
12
14
27

Answers

Answer:

bx8

Step-by-step explanation:

1 yard = 3 feet
6 yards 2 feet = how much feet is 6 yards and 2 feet ?

HELP ASAP

Answers

Answer: 20

Step-by-step explanation: 3 feet in 1 yard, there are 6 yards.

6 x 3 = 18 feet plus the 2 extra = 20 feet

an angle is equal to five times of its complement then what is its measures​

Answers

Answer:

Then the complement of it = 90-x. Given that An angle is equal to 5 times its complement. x = 75. Hence the measure of an angle = 75.

Step-by-step explanation:

What is the value of x?
Enter your answer in the box

Answers

Answer:

The letter "x" is often used in algebra to mean a value that is not yet known. It is called a "variable" or sometimes an "unknown". In x + 2 = 7, x is a variable, but we can work out its value if we try! A variable doesn't have to be "x", it could be "y", "w" or any letter, name or symbol.

Step-by-step explanation:

Rewrite 12/20 with the denominator of 100

Explain how to get it

Answers

60/100, look at picture for work

Which sequence of transformations will map figure Q onto figure Q′?
Two congruent quadrilaterals are shown on a coordinate plane; quadrilateral Q with coordinates negative 9 comma 2, negative 6 comma 4, negative 4 comma 4, and negative 2 comma 2; quadrilateral Q prime with coordinates 2 comma 2, 4 comma 4, 6 comma 4, and 9 comma 2.

A) Translation of (x, y + 2), reflection over x = 1, and 180° rotation about the origin
B) Translation of (x, y − 2), reflection over x = 1, and 180° rotation about the origin
C) Translation of (x, y − 2), reflection over y = 1, and 180° rotation about the origin
D) Translation of (x, y + 2), reflection over y = 1, and 180° rotation about the origin

Answers

Answer:

Which sequence of transformations will map figure Q onto figure Q′?

 Its D.Translation of (x, y + 2), reflection over y = 1, and 180° rotation about the origin

Step-by-step explanation:

The sequence of transformations that will map figure Q onto figure Q′ : 'Translation of (x, y + 2), reflection over y = 1, and 180° rotation about the origin.'

The correct answer is option (D)

What is translation?

"It is a geometric transformation in which the displacement of a figure from one place to another."

What is reflection?

"It is a geometric transformation where all the points of an object are reflected on the line of reflection."

What is rotation?

"It is a transformation in which the object is rotated about a fixed point."

For given question,

Quadrilateral Q with coordinates (-9, 2) ,(-6, 4) ,(- 4, 4) , and (-2, 2) Quadrilateral Q' with coordinates (2, 2), (4, 4), (6, 4), and (9, 2)

Consider a point (-4, 4) of quadrilateral Q.                                                              If we reflect Q over x = 1, then it would push the x coordinate of (-4, 4) to new x value 5 units to the right of x = 1, which is x = 6, and 180 rotation would make x = -6                                                                  But x = -6 is not in Q' .                                                                            So it must be reflection over y = 1.Consider the transformations at option C,                                        First translate to (x, y - 2)                                                                      So, the translation of (-4,4) becomes (-4, 2), reflection over y = 1 gives (-4, 4), and 180° rotation about origin gives      (4, -4)  which is not the coordinate of  Q'Now, consider the translation of (x, y + 2)                                          So, the translation of (-4, 4) becomes (-4, 6), reflection over y = 1 gives (-4, -4), and 180° rotation about origin gives      (4, 4) which is the coordinate of Q'

Therefore, the sequence of transformations that will map figure Q onto figure Q′ : 'Translation of (x, y + 2), reflection over y = 1, and 180° rotation about the origin'

The correct answer is option (D)

Learn more about geometric transformations here:

https://brainly.com/question/15577335

#SPJ2

PLS HELP! ILL MARK BRAINIEST! what is the x and y intercept for this equation?

Answers

x int: (-2, 0) y int: (0,4)

what's the common difference in −1,−4,−7,−10

Answers

Answer:

-3

Step-by-step explanation:

PLEASE PLEASE HELP ME I REALLY NEED HELP

Answers

Answer:

(a) yes it does

(b) the rate is constant because the rate of change (slope) is the same throughout the function.

(c) The rate, or rate of change, is -1/2.

y1 - y2/x1-x2

4 - 2/3 - 9

2/-6

-1/2

Question 13) Olivia has read 143 pages of a 220 page novel. What
percentage of the book does she still need to read? *

Answers

Answer:

77%

Step-by-step explanation:

Answer:

The answer to Question 13 is 35%.

Step-by-step explanation:

Subtract 143 from 220, which will give you 77. Then, you take that 77 and search, "77 out of 220" in Google. You should get 35 and just put the percentage symbol. Olivia still needs to read 35% of the 220 page book:).

Which of the following numbers is irrational?
A) 6.124
B) √7
C) √0.25
D) 0.12¯¯¯¯

Answers

Answer:

B

Step-by-step explanation:

Answer:

B

Step-by-step explanation:

What is the sum of the measures of the exterior angles of this triangle

Answers

Answer:

180°

Step-by-step explanation:

because because that's how much the angle of this triangle is

Answer:  360

The sum of the exterior angles is always 360. This applies to any polygon.

note: If you wanted to sum up the interior angles, then the answer would be 180.

two boats leave a ramp traveling in opposite directions. The second boat is 10 miles per hour faster than the first. After 3 hours they are 150 miles apart. Find the speeds of the boats.

Answers

Answer:

220 mph ; 230 mph

Step-by-step explanation:

Given that :

Speed of First boat = x

Speed of Second boat = y

y = (10 + x) mph

Distance apart after 3 hours = 150 miles

Speed of the boats :

Speed / time = distance

First boat distance = x / 3

Second boat distance = (10+x) /3

x/3 + (10+x)/3 = 150

x + 10 + x = 150 * 3

2x + 10 = 450

2x = 450 - 10

2x = 440

x = 220 mph

Speed of second boat = 220 + 10 = 230 mph

The last answer choice is $0.08 per minute

PLEASE help I'll give 20 points

Answers

i think i figured it out so it’s 0.37 :))))))))

Aubree drives 15 miles in 20 minutes. If she drove three hours in total at the same rate, how far did she go

Answers

135 miles in 3 hours

please help me with this I will name branlist​

Answers

I wish I knew but same profile pic pog

Answer:

IT's -3.6 & 14.4 so D is the correct answer

Step-by-step explanation:

x= -3.6 y= 14.4

HOW I SOLVED IT

-3.6 x 3 = -10.8    2 x 14.4 = 28.8 - 10.8 = 18  so correct since it must equal to 18

-2/3 x -3.6 =2.4  + 12 = 14.4 so correct since y = 14.4

Brainilest would be appreciated :)

Helppppp pleazeee!!!

Answers

I believe the answer is Chord AB

Answer:

wooo

Step-by-step explanation:

70% off
original price!
Abdul wants to buy a cat calendar. The original price is $5.30. What is the sale price?
I need this worked out please

Answers

Answer:

the price with the offer is going to be 1.59$

Step-by-step explanation:

[tex] \frac{x}{5.30} = \frac{70}{100} [/tex]

[tex]100(x) = 70(5.30)[/tex]

[tex]100(x) = 371[/tex]

[tex] \frac{100(x)}{100} = \frac{371}{100} [/tex]

[tex]x = 3.71[/tex]

[tex]70\% \: \: \: of \: \: \: 5.30 \: \: \: is \: \: \: 3.71[/tex]

[tex]5.30 - 3.71 = 1.59[/tex]

zasxedcrfvtgbyhjffgh...

Answers

Answer:

yes?

Step-by-step explanation:

what is y<-x+5 and y≤ 2x-1??

Answers

9514 1404 393

Answer:

  see attached

Step-by-step explanation:

The pair of inequalities is graphed in the attachment.

The first inequality graphs as a shaded area below the dashed line y=-x+5.

The second inequality graphs as a shaded area below the solid line y=2x-1.

The solution set is the doubly-shaded area below the point where the lines intersect.

Matt gets $0.10 every 2 days
How much does he get in 6 days?

Answers

Answer:

$0.30

Step-by-step explanation:

2 days = $0.10

6 days = [tex]\frac{6}{2}[/tex] × $0.10

           = $0.30

Other Questions
When save a newly created template, you use which extension for the file type? .doc, .docx, .doct, .dotx? I need help with this question.Pls help. Why is it significant that Elizabeth Proctor would not tell the court of her husbands infidelity? What Egyptian Kingdom built the pyramids? * HELP ASAP multiple question right answer only!! (not all of the above) need help please help Which statement explains one way that ponds and lakes differ? Ponds are shallower than lakes. Ponds are larger than lakes. Pond water moves more than lake water. Lakes have more plants at the bottom than ponds. HELP WHATS THE ANSWER what is the representative organism for sponges??Worth 20 points!!! help im being TIMED A batter hits two balls during practice. Ball A travels 10 meters, while ball B travels 12 meters. The batter decides to hit ball A with less force. How will this change the motion of ball B? The motion of ball B will increase. The motion of ball B will decrease. The motion of ball B will stay the same. The motion of ball B will become the same as ball A. For every | | followers Matthew has, Kevin has | |followers.PLEASE HELP ILL MARK BRAINLEST i need someone to help me with my algebra homework :( You have certain undeniable rights because you are alive and human including (like right to life)that are protected by the Bill of Rights. Which principle does this reflect?A. Individual rights B. Popular Sovereignty C.FederalismD. Separation of Powers if you have 6 pairs of homologous chromosomes. how many autosomes do you have? If there are oncoming cars to the left of you and a child on a bike to the right, what is the correct action to take The commitment of the United States government to the principles of free enterprisecontributed to -A government owning of some businessesB increasing limits on consumer choiceC less government interaction in the economyD government running of westward expansion I need to know good world languages to study. Camilla has already baked five pies and she can make one pie with additional cup of sugar she buys how many additional cups of sugar does Camila need in order to bake a total of 49 pies can u pls help me with this question Which statements are true about the ordered pair (10, 5) and the system of equations?{ 2x5y=5{ x+2y=11Select each correct answer.The ordered pair (10, 5) is a solution to the first equation because it makes the first equation true.The ordered pair (10, 5) is a solution to the second equation because it makes the second equation true.The ordered pair (10, 5) is not a solution to the system because it makes at least one of the equations false.The ordered pair (10, 5) is a solution to the system because it makes both equations true.